Med Surg GI/GU Diabetes Questions

¡Supera tus tareas y exámenes ahora con Quizwiz!

The nurse notes that the HbA1c level of an assigned patient demonstrated a drop from 9.4% to 5.4%. What can the nurse infer from these findings? a. The patient's blood glucose control has improved over the last several months. b. The patient has been less compliant with the prescribed treatment regimen. c. The patient is experiencing a reduction in insulin sensitivity. d. The patient has less need for insulin.

ANS: A The patient's blood glucose control has improved over the last several months. HbA1c is a diagnostic assessment used to review blood glucose levels retrospectively. A reduction in the value indicates improved glucose control by the patient. There is no evidence of insulin sensitivity. The need for insulin is not decreased in this patient.

The nurse is educating a 50-year-old patient about diabetes monitoring. Which statement reinforces the American Diabetes Association's (ADA's) recommendation? a. Obtain regularly scheduled fasting blood glucose levels. b. Strictly adhere to weight reduction diets. c. Exercise regularly in intervals lasting a minimum of 30 minutes. d. Use stress reduction techniques.

ANS: A Obtain regularly scheduled fasting blood glucose levels. The ADA recommends screening with a fasting blood glucose. Adherence to a weight loss plan, regular exercise, and stress reduction techniques help control diabetes but do not monitor it.

Which goal is the primary objective of a diabetic diet? a. Adequate nutrition with weight control b. Exclusion of all sweets c. Increased fat intake for greater energy e. Elimination of all fast foods

ANS: A Adequate nutrition with weight control Currently, the diabetic diet is much less stringent than diets of years past. The primary goal of the current diabetic diet includes adequate nutrition with weight and cholesterol control. The newer diets allow for some sweets and some fast foods. Fats are not adequate sources of energy. Fat intake should be limited to reduce complications related to weight gain and cardiovascular concerns.

The nurse is counseling an overweight, noncompliant, 30-year-old female with type 2 diabetes. Which change is most important for the nurse to suggest? a. Begin an exercise program and lose weight. b. Obtain annual eye examinations. c. Keep a food diary. d. Inspect feet daily.

ANS: A Begin an exercise program and lose weight. All of these changes are important, but exercise and weight loss are priority changes. In the type 2 diabetic, weight reduction and increased physical activity can restore blood glucose to normal levels and maintained it—hence the importance of diet and exercise in the management of type 2 diabetes. Annual eye examinations are important to detect onset of diabetic retinopathy. A food diary can help the patient to visualize food intake that may be subconscious otherwise. Diabetics are prone to foot problems and wounds and should inspect their feet daily.

A nurse assesses clients who are at risk for diabetes mellitus. Which client is at greatest risk? a. A 29-year-old Caucasian b. A 32-year-old African-American c. A 44-year-old Asian d. A 48-year-old American Indian

ANS: D d. A 48-year-old American Indian Diabetes is a particular problem among African Americans, Hispanics, and American Indians. The incidence of diabetes increases in all races and ethnic groups with age. Being both an American Indian and middle-aged places this client at highest risk.

A nurse teaches a client who is prescribed an insulin pump. Which statement should the nurse include in this clients discharge education? a. Test your urine daily for ketones. b. Use only buffered insulin in your pump. c. Store the insulin in the freezer until you need it. d. Change the needle every 3 days.

ANS: D d. Change the needle every 3 days. Having the same needle remain in place through the skin for longer than 3 days drastically increases the risk for infection in or through the delivery system. Having an insulin pump does not require the client to test for ketones in the urine. Insulin should not be frozen. Insulin is not buffered.

A teaching plan for a patient with diabetes is focused on smoking cessation and the control of hypertension for the avoidance of microvascular complications. What are examples of microvascular complications? (Select all that apply.) a. Macular degeneration b. End-stage renal disease (ESRD) c. Coronary artery disease (CAD) d. Peripheral vascular disease (PVD) e. Cerebrovascular accident (CVA)

ANS: A, B Macular degeneration and ESRD are both microvascular complications. CAD, PVD, and CVA are all macrovascular complications.

Which factor(s) may cause diabetes mellitus (DM)? (select all that apply.) a. Genetic b. Microbiologic c. Metabolic d. Allogenic e. Immunologic

ANS: A, B, C, E a. Genetic b. Microbiologic c. Metabolic e. Immunologic Genetic, microbiologic, metabolic, and immunologic factors influence the development of diabetes mellitus. Allogenic refers to cells or tissues that are from different individuals in the same species.

A nurse collaborates with the interdisciplinary team to develop a plan of care for a client who is newly diagnosed with diabetes mellitus. Which team members should the nurse include in this interdisciplinary team meeting? (Select all that apply.) a. Registered dietitian b. Clinical pharmacist c. Occupational therapist d. Health care provider e. Speech-language pathologist

ANS: A, B, D When planning care for a client newly diagnosed with diabetes mellitus, the nurse should collaborate with a registered dietitian, clinical pharmacist, and health care provider. The focus of treatment for a newly diagnosed client would be nutrition, medication therapy, and education. The nurse could also consult with a diabetic educator. There is no need for occupational therapy or speech therapy at this time.

The nurse is caring for a patient suspected of having ketoacidosis. Which manifestation(s) is/are characteristic with early ketoacidosis? (select all that apply.) a. Fruity breath b. Polyuria c. Nausea d. Thirst e. Sunken eyes

ANS: A, B, D a. Fruity breath b. Polyuria d. Thirst Ketoacidosis is a complication associated with type 1 diabetes. Some of the earliest symptoms may be polyuria, fatigue, anorexia, abdominal pain, and a fruity smell to the breath. Later signs and symptoms include sunken eyes as a result of excessive dehydration. Nausea is not associated with ketoacidosis.

How is the Whipple triad described? (Select all that apply.) a. Symptoms of hypoglycemia are present. b. Low blood glucose levels are documented when symptoms are present. c. Symptoms can be reproduced with an injection of regular insulin, 10 units. d. Muscular activity does not have any effect on blood glucose. e. Symptoms improved when the blood glucose level rises.

ANS: A, B, E Whipple triad is the presence of the symptoms of: - hypoglycemia (e.g., diaphoresis, pallor, tachycardia) - the documentation of low blood glucose levels when symptoms are present - the improvement of symptoms as the blood glucose level rises.

After teaching a client with type 2 diabetes mellitus, the nurse assesses the clients understanding. Which statement made by the client indicates a need for additional teaching? a. I need to have an annual appointment even if my glucose levels are in good control. b. Since my diabetes is controlled with diet and exercise, I must be seen only if I am sick. c. I can still develop complications even though I do not have to take insulin at this time. d. If I have surgery or get very ill, I may have to receive insulin injections for a short time.

ANS: B Clients with diabetes need to be seen at least annually to monitor for long-term complications, including visual changes, microalbuminuria, and lipid analysis. The client may develop complications and may need insulin in the future.

After teaching a young adult client who is newly diagnosed with type 1 diabetes mellitus, the nurse assesses the clients understanding. Which statement made by the client indicates a correct understanding of the need for eye examinations? a. At my age, I should continue seeing the ophthalmologist as I usually do. b. I will see the eye doctor when I have a vision problem and yearly after age c. My vision will change quickly. I should see the ophthalmologist twice a year. d. Diabetes can cause blindness, so I should see the ophthalmologist yearly.

ANS: D d. Diabetes can cause blindness, so I should see the ophthalmologist yearly. Diabetic retinopathy is a leading cause of blindness in North America. All clients with diabetes, regardless of age, should be examined by an ophthalmologist (rather than an optometrist or optician) at diagnosis and at least yearly thereafter.

The nurse is explaining the underlying pathophysiology of type 1 diabetes to a newly diagnosed patient. Which information accurately explains why the type 1 diabetic does not produce adequate insulin? a. A pituitary disorder inhibits beta cells. b. An allergic response alters beta cell responses to hyperglycemia. c. Alpha cells proliferated in the islets of Langerhans. d. The body's immune system destroyed beta cells.

ANS: D. The body's immune system destroyed beta cells. In type 1 diabetes mellitus (DM), the beta cells on the islets of Langerhans are destroyed by an autoimmune reaction.

A patient with type 1 diabetes has told the nurse that his most recent urine test for ketones was positive. What is the nurses most plausible conclusion based on this assessment finding? a. The patient should withhold his next scheduled dose of insulin. b. The patient should promptly eat some protein and carbohydrates. c. The patients insulin levels are inadequate. d. The patient would benefit from a dose of metformin (Glucophage).

Ans: C The patients insulin levels are inadequate. Feedback: Ketones in the urine signal that there is a deficiency of insulin and that control of type 1 diabetes is deteriorating. Withholding insulin or eating food would exacerbate the patients ketonuria. Metformin will not cause short-term resolution of hyperglycemia.

A diabetes nurse educator is presenting the American Diabetes Association (ADA) recommendations for levels of caloric intake. What do the ADAs recommendations include? A) 10% of calories from carbohydrates, 50% from fat, and the remaining 40% from protein B) 10% to 20% of calories from carbohydrates, 20% to 30% from fat, and the remaining 50% to 60% from protein C) 20% to 30% of calories from carbohydrates, 50% to 60% from fat, and the remaining 10% to 20% from protein D) 50% to 60% of calories from carbohydrates, 20% to 30% from fat, and the remaining 10% to 20% from protein

Ans: D 50% to 60% of calories from carbohydrates, 20% to 30% from fat, and the remaining 10% to 20% from protein Feedback: Currently, the ADA and the Academy of Nutrition and Dietetics (formerly the American Dietetic Association) recommend that for all levels of caloric intake, 50% to 60% of calories come from carbohydrates, 20% to 30% from fat, and the remaining 10% to 20% from protein.

A patient has received a diagnosis of type 2 diabetes. The diabetes nurse has made contact with the patient and will implement a program of health education. What is the nurses priority action? A) Ensure that the patient understands the basic pathophysiology of diabetes. B) Identify the patients body mass index. C) Teach the patient survival skills for diabetes. D) Assess the patients readiness to learn.

Ans: D Assess the patients readiness to learn. Before initiating diabetes education, the nurse assesses the patients (and familys) readiness to learn. This must precede other physiologic assessments (such as BMI) and providing health education.

A nurse assesses a client who has diabetes mellitus and notes the client is awake and alert, but shaky, diaphoretic, and weak. Five minutes after administering a half-cup of orange juice, the clients clinical manifestations have not changed. Which action should the nurse take next? a. Administer another half-cup of orange juice. b. Administer a half-ampule of dextrose 50% intravenously. c. Administer 10 units of regular insulin subcutaneously. d. Administer 1 mg of glucagon intramuscularly.

a. Administer another half-cup of orange juice. This client is experiencing mild hypoglycemia. For mild hypoglycemic manifestations, the nurse should administer oral glucose in the form of orange juice. If the symptoms do not resolve immediately, the treatment should be repeated. The client does not need intravenous dextrose, insulin, or glucagon.

A nurse is formulating a teaching plan for a 22-year-old woman taking rosiglitazone (Avandia). What should the nurse include information about in this plan to caution this patient? a. Decreased effectiveness of her birth control pills b. Excessive exposure to the sun c. Sudden drop in blood pressure with dizziness d. Possible severe diarrhea

a. Decreased effectiveness of her birth control pills Avandia causes some birth control pills to be less effective.

What should a nurse include when drawing up a patient's diabetes teaching plan? a. Develop an exercise plan because regular exercise helps control blood glucose levels. b. Monitor blood sugar levels only if not feeling well to ensure that the fingertips are not pricked too much. c. If nervousness, palpitations, or hunger is experienced, take a small dose (1 to 2 U) of regular insulin and call the physician. d. Use over-the-counter measures for any foot blisters, calluses, or wounds before seeking medical help.

a. Develop an exercise plan because regular exercise helps control blood glucose levels. Exercise is an integral part of the patient's ability to take charge of his or her diabetes and needs to be included in the teaching plan.

A nurse assigned to care for a patient with diabetic ketoacidosis (DKA) is aware that this is a life-threatening condition. What will DKA result in? a. Disorder of carbohydrates, fats, and proteins metabolism b. Storage of glycogen, resulting in a severe shortage of glucose in the bloodstream c. Dangerously elevated pH and bicarbonate levels in the blood d. Severe hypoglycemia, which can result in coma and convulsions

a. Disorder of carbohydrates, fats, and proteins metabolism DKA is mainly related to the use of fat as an energy source because of an inability of the body to use glucose. The metabolism of fat produces ketones.

A preoperative nurse assesses a client who has type 1 diabetes mellitus prior to a surgical procedure. The clients blood glucose level is 160 mg/dL. Which action should the nurse take? a. Document the finding in the clients chart. b. Administer a bolus of regular insulin IV. c. Call the surgeon to cancel the procedure. d. Draw blood gases to assess the metabolic state.

a. Document the finding in the clients chart. Clients who have type 1 diabetes and are having surgery have been found to have fewer complications, lower rates of infection, and better wound healing if blood glucose levels are maintained at between 140 and 180 mg/dL throughout the perioperative period. The nurse should document the finding and proceed with other operative care. The need for a bolus of insulin, canceling the procedure, or drawing arterial blood gases is not required.

A nurse prepares to administer insulin to a client at 1800. The clients medication administration record contains the following information: Insulin glargine: 12 units daily at 1800 Regular insulin: 6 units QID at 0600, 1200, 1800, 2400 Based on the clients medication administration record, which action should the nurse take? a. Draw up and inject the insulin glargine first, and then draw up and inject the regular insulin. b. Draw up and inject the insulin glargine first, wait 20 minutes, and then draw up and inject the regular insulin. c. First draw up the dose of regular insulin, then draw up the dose of insulin glargine in the same syringe, mix, and inject the two insulins together. d. First draw up the dose of insulin glargine, then draw up the dose of regular insulin in the same syringe, mix, and inject the two insulins together.

a. Draw up and inject the insulin glargine first, and then draw up and inject the regular insulin. Insulin glargine must not be diluted or mixed with any other insulin or solution. Mixing results in an unpredictable alteration in the onset of action and time to peak action. The correct instruction is to draw up and inject first the glargine and then the regular insulin right afterward.

A nurse suspects that a patient with type 1 diabetes may be experiencing the Somogyi phenomenon. What symptom supports this suspicion? a. Headache on awakening and enuresis b. 6 AM blood sugar of 58 mg/dL and nausea c. Abdominal pain and elevated blood pressure d. Drowsiness and disorientation after eating

a. Headache on awakening and enuresis The Somogyi phenomenon occurs because of a rebound hyperglycemia after a period of hypoglycemia during the early morning. The patient wakes with a headache, enuresis, nausea and vomiting, nightmares, and a high level of blood sugar.

A nurse provides diabetic education at a public health fair. Which disorders should the nurse include as complications of diabetes mellitus? (Select all that apply.) a. Stroke b. Kidney failure c. Blindness d. Respiratory failure e. Cirrhosis

a. Stroke b. Kidney failure c. Blindness Complications of diabetes mellitus are caused by macrovascular and microvascular changes. Macrovascular complications include coronary artery disease, cerebrovascular disease, and peripheral vascular disease. Microvascular complications include nephropathy, retinopathy, and neuropathy. Respiratory failure and cirrhosis are not complications of diabetes mellitus.

A nurse prepares to administer prescribed regular and NPH insulin. Place the nurses actions in the correct order to administer these medications. 1. Inspect bottles for expiration dates. 2. Gently roll the bottle of NPH between the hands. 3. Wash your hands. 4. Inject air into the regular insulin. 5. Withdraw the NPH insulin. 6. Withdraw the regular insulin. 7. Inject air into the NPH bottle. 8. Clean rubber stoppers with an alcohol swab. a. 1, 3, 8, 2, 4, 6, 7, 5 b. 3, 1, 2, 8, 7, 4, 6, 5 c. 8, 1, 3, 2, 4, 6, 7, 5 d. 2, 3, 1, 8, 7, 5, 4, 6

b. 3, 1, 2, 8, 7, 4, 6, 5 After washing hands, it is important to inspect the bottles and then to roll the NPH to mix the insulin. Rubber stoppers should be cleaned with alcohol after rolling the NPH and before sticking a needle into either bottle. It is important to inject air into the NPH bottle before placing the needle in a regular insulin

A nurse reviews laboratory results for a client with diabetes mellitus who is prescribed an intensified insulin regimen: Fasting blood glucose: 75 mg/dL Postprandial blood glucose: 200 mg/dL Hemoglobin A1c level: 5.5% How should the nurse interpret these laboratory findings? a. Increased risk for developing ketoacidosis b. Good control of blood glucose c. Increased risk for developing hyperglycemia d. Signs of insulin resistance

b. Good control of blood glucose The client is maintaining blood glucose levels within the defined ranges for goals in an intensified regimen. Because the clients glycemic control is good, he or she is not at higher risk for ketoacidosis or hyperglycemia and is not showing signs of insulin resistance.

A patient has come into the emergency department accompanied by a friend who states that the patient had been acting very strangely and seems confused. The friend states that the patient has diabetes and takes insulin. Which signs of hypoglycemia might the nurse assess? a. Slow pulse rate and low blood pressure b. Irritability, anxiety, confusion, and dizziness c. Flushing, anger, and forgetfulness d. Sleepiness, edema, and sluggishness

b. Irritability, anxiety, confusion, and dizziness When blood sugar levels fall, hormones are activated to increase serum glucose. One of the hormones is epinephrine, which causes these symptoms.

A nurse reviews the medication list of a client with a 20-year history of diabetes mellitus. The client holds up the bottle of prescribed duloxetine (Cymbalta) and states, My cousin has depression and is taking this drug. Do you think Im depressed? How should the nurse respond? a. Many people with long-term diabetes become depressed after a while. b. Its for peripheral neuropathy. Do you have burning pain in your feet or hands? c. This antidepressant also has anti-inflammatory properties for diabetic pain. d. No. Many medications can be used for several different disorders.

b. Its for peripheral neuropathy. Do you have burning pain in your feet or hands? Damage along nerves causes peripheral neuropathy and leads to burning pain along the nerves. Many drugs, including duloxetine (Cymbalta), can be used to treat peripheral neuropathy. The nurse should assess the client for this condition and then should provide an explanation of why this drug is being used. This medication, although it is used for depression, is not being used for that reason in this case. Duloxetine does not have anti-inflammatory properties. Telling the client that many medications are used for different disorders does not provide the client with enough information to be useful.

A home health care nurse is assessing a patient with type 1 diabetes who has been controlled for 6 months. The nurse is surprised and concerned about a blood glucose reading of 52 mg/dL. What action by this patient most likely caused this episode of hypoglycemia? a. Taking a new form of birth control pill this morning b. Using large amounts of sugar substitute in her tea this morning c. A 2-hour long exercise class at the spa this morning d. Administering an insufficient dose of insulin this morning

c. A 2-hour long exercise class at the spa this morning Excessive exercise used up the glucose that was made available by the insulin taken by the patient. The patient now has too much insulin for the available glucose and has become hypoglycemic.

A young patient complains that diabetes is causing her to "have no life at all. It's too hard." What is the most helpful response by the nurse? a. "Yes, you must make some sacrifices." b. "It's hard, but with significant alterations in your lifestyle, you can live a long life." c. "What's hard about exercise, diet, and medicine?" d. "Let's talk about what makes it so hard."

d. "Let's talk about what makes it so hard." Involving the patient in decisions about how she will cope with her diabetes will make the goals more realistic and personal, which will give her a greater chance of success in meeting them.

A patient comes to the diabetes clinic and confides to the nurse that she does not follow the diet exchange program that she was given. What is the best response by the nurse? a. "The exchange program is a carefully developed and very important program that allows you to take control of your disease." b. "A lot of people have trouble with that program. You aren't the first one to go off your diet." c. "We had better check your blood work to see what you've done to yourself." d. "Okay. Let's talk about what you do eat and drink and how you manage your diabetes."

d. "Okay. Let's talk about what you do eat and drink and how you manage your diabetes." To evaluate the effectiveness of treatment, the nurse must first find out how the patient perceives the importance of diet, drugs, and exercise.

How long does it take for Humulin R 20 units to peak? a. 15 minutes b. 30 minutes c. 1 hour d. 2 hours

d. 2 hours Humulin R has its onset in approximately 30 minutes, but its peak is in 2 hours.

The self-care goal of a patient with diabetes is to keep the blood sugar within normal limits. What causes hyperglycemia to occur? a. Blood glucose levels rise, stimulating the production of insulin. b. Insulin conversion of glycogen to glucose is inhibited. c. The body responds to glucose-starved tissues by changing stored glycogen into glucose. d. Glycogen is unable to be stored in the liver and muscles.

c. The body responds to glucose-starved tissues by changing stored glycogen into glucose. The hypothalamus is receiving a message that the cells need glucose, so it responds by adding more glucose to the already overburdened blood.

A nurse assesses a client with diabetes mellitus 3 hours after a surgical procedure and notes the clients breath has a fruity odor. Which action should the nurse take? a. Encourage the client to use an incentive spirometer. b. Increase the clients intravenous fluid flow rate. c. Consult the provider to test for ketoacidosis. d. Perform meticulous pulmonary hygiene care.

c. Consult the provider to test for ketoacidosis. The stress of surgery increases the action of counterregulatory hormones and suppresses the action of insulin, predisposing the client to ketoacidosis and metabolic acidosis. One manifestation of ketoacidosis is a fruity odor to the breath. Documentation should occur after all assessments have been completed. Using an incentive spirometer, increasing IV fluids, and performing pulmonary hygiene will not address this clients problem.

A nurse explains that type 1 diabetes mellitus is a disease in which the body does not produce enough insulin. What is the reason that the blood glucose is elevated? a. Prolonged elevation of stress hormone (cortisol, epinephrine, glucagon, growth hormone) levels b. Malfunction of the glycogen-storing capabilities of the liver c. Destruction of the beta cells in the pancreas d. Insulin resistance of the receptor cells in the muscle tissue

c. Destruction of the beta cells in the pancreas Type 1 diabetes mellitus is a disease in which the pancreas does not produce adequate insulin because of the destruction of beta cells.

What has most likely occurred in a patient who has been diagnosed with endogenous hypoglycemia? a. Taken an overdose of hypoglycemic drugs b. Been following a very restricted fasting diet or is malnourished c. Excessive secretion of insulin or an increase in glucose metabolism d. Exercised unwittingly without replenishing needed fluids and nutrients

c. Excessive secretion of insulin or an increase in glucose metabolism Endogenous refers to within; in this patient, it refers to internal factors, such as an increase of insulin or glucose metabolism. Both conditions would lead to hypoglycemia.

As part of a teaching plan in preparation for discharge, a patient with type 1 diabetes needs guidelines for exercise. Which guideline should be included? a. Plan exercise so that it coincides with the peak action of insulin. b. Insulin should be injected into the lower extremity before exercise because that site provides the greatest absorption. c. Exercise should be performed daily at the same time of day and at the same intensity. d. Keep exercise at a minimum to conserve your energy.

c. Exercise should be performed daily at the same time of day and at the same intensity. If the body is using more glucose than available, the body will draw on fatty acids, which will give off ketones.

A patient tells a nurse that she eats "huge" amounts of food but stays hungry most of the time. What should the nurse explain as the cause of hunger experienced by persons with type 1 diabetes? a. Excess amount of glucose b. Need for additional calories to correct the increased metabolism c. Fact that the cells cannot use the blood glucose d. Need for exercise to stimulate insulin secretion

c. Fact that the cells cannot use the blood glucose The cells cannot use the glucose without insulin, so the patient with diabetes still feels hungry even though abundant glucose is circulating in the blood.

A patient with type 2 diabetes shows a blood sugar reading of 68 at 6 AM. What action should the nurse implement based on the reading of 72 mg/dL? a. Notify the charge nurse of the reading. b. Give regular insulin per a sliding scale. c. Give him 8 oz of skim milk. d. Administer the oral glucose tablet.

c. Give him 8 oz of skim milk. The patient is hypoglycemic and needs an immediate source of glucose, such as milk or orange juice. The oral hypoglycemic agent will not work quickly enough. The charge nurse can be notified later. Giving insulin per a sliding scale would lower the blood sugar level.

A nurse assesses a client who is being treated for hyperglycemic-hyperosmolar state (HHS). Which clinical manifestation indicates to the nurse that the therapy needs to be adjusted? a. Serum potassium level has increased. b. Blood osmolarity has decreased. c. Glasgow Coma Scale score is unchanged. d. Urine remains negative for ketone bodies.

c. Glasgow Coma Scale score is unchanged. A slow but steady improvement in central nervous system functioning is the best indicator of therapy effectiveness for HHS. Lack of improvement in the level of consciousness may indicate inadequate rates of fluid replacement. The Glasgow Coma Scale assesses the clients state of consciousness against criteria of a scale including best eye, verbal, and motor responses. An increase in serum potassium, decreased blood osmolality, and urine negative for ketone bodies do not indicate adequacy of treatment.

After teaching a client who is newly diagnosed with type 2 diabetes mellitus, the nurse assesses the clients understanding. Which statement made by the client indicates a need for additional teaching? a. I should increase my intake of vegetables with higher amounts of dietary fiber. b. My intake of saturated fats should be no more than 10% of my total calorie intake. c. I should decrease my intake of protein and eliminate carbohydrates from my diet. d. My intake of water is not restricted by my treatment plan or medication regimen.

c. I should decrease my intake of protein and eliminate carbohydrates from my diet. The client should not completely eliminate carbohydrates from the diet, and should reduce protein if microalbuminuria is present. The client should increase dietary intake of complex carbohydrates, including vegetables, and decrease intake of fat. Water does not need to be restricted unless kidney failure is present.

What does the lack of insulin in patients with type 1 diabetes cause that increases the risk for cardiovascular disorders? a. High glucose levels that irritate and shrink the vessels b. Inadequate metabolism of proteins, which causes ketosis c. Increased fatty acid levels d. Increased metabolism of ketones, which causes hypertension

c. Increased fatty acid levels The increase in fatty acid levels causes an increase in the level of triglycerides and an attendant rise in low-density lipoprotein levels.

A patient with type 1 diabetes asks why his 0700 insulin has been changed from NPH insulin to 70/30 premixed insulin. What is the best explanation by the nurse that explains about 70/30 insulin mixture? a. It is absorbed more rapidly into the bloodstream. b. It has no peak action time and lasts all day. c. It makes insulin administration easier and safer. d. It provides a bolus of rapid-acting insulin to prevent hyperglycemia after breakfast.

c. It makes insulin administration easier and safer. 70/30 insulin is 30% rapid-acting insulin and 70% intermediate-acting insulin. The rapid action of the 0700 premixed insulin prevents hyperglycemia after the morning meal and the mixed drug reduces the risk of error in drawing up two insulins.

A nurse cares for a client experiencing diabetic ketoacidosis who presents with Kussmaul respirations. Which action should the nurse take? a. Administration of oxygen via face mask b. Intravenous administration of 10% glucose c. Implementation of seizure precautions d. Administration of intravenous insulin

d. Administration of intravenous insulin The rapid, deep respiratory efforts of Kussmaul respirations are the bodys attempt to reduce the acids produced by using fat rather than glucose for fuel. Only the administration of insulin will reduce this type of respiration by assisting glucose to move into cells and to be used for fuel instead of fat. The client who is in ketoacidosis may not experience any respiratory impairment and therefore does not need additional oxygen. Giving the client glucose would be contraindicated. The client does not require seizure precautions.

A nurse reviews the laboratory results of a client who is receiving intravenous insulin. Which should alert the nurse to intervene immediately? a. Serum chloride level of 98 mmol/L b. Serum calcium level of 8.8 mg/dL c. Serum sodium level of 132 mmol/L d. Serum potassium level of 2.5 mmol/L

d. Serum potassium level of 2.5 mmol/L Insulin activates the sodium-potassium ATPase pump, increasing the movement of potassium from the extracellular fluid into the intracellular fluid, resulting in hypokalemia. In hyperglycemia, hypokalemia can also result from excessive urine loss of potassium. The chloride level is normal. The calcium and sodium levels are slightly low, but this would not be related to hyperglycemia and insulin administration.

A nurse teaches a client with type 1 diabetes mellitus. Which statement should the nurse include in this clients teaching to decrease the clients insulin needs? a. Limit your fluid intake to 2 liters a day. b. Animal organ meat is high in insulin. c. Limit your carbohydrate intake to 80 grams a day. d. Walk at a moderate pace for 1 mile daily.

d. Walk at a moderate pace for 1 mile daily. Moderate exercise such as walking helps regulate blood glucose levels on a daily basis and results in lowered insulin requirements for clients with type 1 diabetes mellitus. Restricting fluids and eating organ meats will not reduce insulin needs. People with diabetes need at least 130 grams of carbohydrates each day.

A nurse reviews the medication list of a client recovering from a computed tomography (CT) scan with IV contrast to rule out small bowel obstruction. Which medication should alert the nurse to contact the provider and withhold the prescribed dose? a. Pioglitazone (Actos) b. Glimepiride (Amaryl) c. Glipizide (Glucotrol) d. Metformin (Glucophage)

d. Metformin (Glucophage) Glucophage should not be administered when the kidneys are attempting to excrete IV contrast from the body. This combination would place the client at high risk for kidney failure. The nurse should hold the metformin dose and contact the provider. The other medications are safe to administer after receiving IV contrast.

A nurse cares for a client who has type 1 diabetes mellitus. The client asks, Is it okay for me to have an occasional glass of wine? How should the nurse respond? a. Drinking any wine or alcohol will increase your insulin requirements. b. Because of poor kidney function, people with diabetes should avoid alcohol. c. You should not drink alcohol because it will make you hungry and overeat. d. One glass of wine is okay with a meal and is counted as two fat exchanges.

d. One glass of wine is okay with a meal and is counted as two fat exchanges. Under normal circumstances, blood glucose levels will not be affected by moderate use of alcohol when diabetes is well controlled. Because alcohol can induce hypoglycemia, it should be ingested with or shortly after a meal. One alcoholic beverage is substituted for two fat exchanges when caloric intake is calculated. Kidney function is not impacted by alcohol intake. Alcohol is not associated with increased hunger or overeating.

A nurse teaches a client with diabetes mellitus who is experiencing numbness and reduced sensation. Which statement should the nurse include in this clients teaching to prevent injury? a. Examine your feet using a mirror every day. b. Rotate your insulin injection sites every week. c. Check your blood glucose level before each meal. d. Use a bath thermometer to test the water temperature.

d. Use a bath thermometer to test the water temperature. Clients with diminished sensory perception can easily experience a burn injury when bathwater is too hot. Instead of checking the temperature of the water by feeling it, they should use a thermometer. Examining the feet daily does not prevent injury, although daily foot examinations are important to find problems so they can be addressed. Rotating insulin and checking blood glucose levels will not prevent injury.

What diabetes occurs during pregnancy?

gestational diabetes

Which laboratory values are consistent with a patient in ketoacidosis? a. Blood urea nitrogen (BUN) of 35 mg/dL b. Carbon dioxide (CO2) of 40 mEq/L c. pH of 7.54 d. Blood glucose of 70 mg/dL

ANS: A Blood urea nitrogen (BUN) of 35 mg/dL Diabetic ketoacidosis results when the body attempts to metabolize protein and fats, which results in high BUN readings. The CO2 should be normal or low depending on the effectiveness of Kussmaul respirations. The arterial pH will be low, and there will be high glucose, which the diabetic patient cannot use.

The nurse is educating the patient about the significance of islet cell antibodies. Which statement accurately describes islet cell antibodies? a. Islet cell antibodies cause beta cells to quit producing insulin and lead to type 1 diabetes mellitus (DM). b. Islet cell antibodies protect beta cells from viral attack. c. Islet cell antibodies increase production of insulin from beta cells. d. Islet cell antibodies decrease the size of the pancreas.

ANS: A Islet cell antibodies cause beta cells to quit producing insulin and lead to type 1 diabetes mellitus (DM). The antibodies cause beta cells to quit production of insulin.

The nurse is caring for a patient who struggles to maintain glycemic control at night and during early morning hours. Which statement correctly explains the reason for this problem? a. Counter regulatory hormones produce hyperglycemia. b. Hyperglycemia of dawn phenomenon does not react to insulin. c. Hypoglycemia quickly follows the dawn phenomenon. d. Food intake fails to change hyperglycemia of dawn phenomenon.

ANS: A Counter regulatory hormones produce hyperglycemia. Dawn phenomenon is produced in the morning by the circadian release of growth hormones, epinephrine, and glucagon during the night. Rebound hyperglycemia, also known as the Somogyi effect, follows a period of hypoglycemia, often during sleep. When hypoglycemia occurs, the body secretes glucagon, epinephrine, growth hormone, and cortisol to counteract the effects of low blood sugar. The patient may report nightmares and night sweats along with morning elevated serum glucose; if the patient increases the insulin dose, it worsens the problem. The dawn phenomenon is characterized by elevated blood glucose in the morning and is caused by release of growth hormone, glucagon, and epinephrine during the night, as part of the body's natural circadian rhythm. These hormones act to raise the body's blood sugar. The dawn phenomenon is the reason why most people with diabetes do not tolerate carbohydrates well in the morning. The treatment is an intermediate-acting insulin at night.

A long-term diabetic patient reports that he has been diagnosed with early cardiovascular disease. How does diabetes predispose the patient to cardiovascular complications? a. Hyperglycemic periods cause thickening of the basement membrane in vessels, which causes atherosclerosis. b. Hypoglycemic periods increase cortisol release, which causes hypertension. c. Insulin constricts the cardiovascular vessels, which causes congestive heart failure. d. Diabetes decrease in the body's ability to digest fats by the pancreas, which leads to increased coronary artery blockage.

ANS: A Hyperglycemic periods cause thickening of the basement membrane in vessels, which causes atherosclerosis. Periods of hyperglycemia cause thickening of the vessels, chiefly the basement membrane (thin layer of connective tissue under the epithelium). The vessels of the retina, renal glomeruli, peripheral nerves, muscles, and skin are affected. Larger vessels are also affected, predisposing the patient to atherosclerosis and vascular occlusion. Two out of three people with diabetes die prematurely from heart attack or stroke.

The nurse is reviewing the patient's prescribed insulin regimen. The nurse notes that the physician has ordered a long-lasting insulin. Which medication best meets this criteria? a. Lantus b. NovoLog c. Humalog d. Regular

ANS: A Lantus Lantus is a long-lasting insulin. It may be administered only one time per day. NovoLog and Humalog are both rapid-onset insulin preparations. Regular insulin is classified as a short-acting insulin.

A patient asks the nurse if stress can be a potential cause of type 2 diabetes. Which response is most appropriate for the nurse to make? a. "Stress decreases the number of alpha cells in the pancreas, and increases the workload on the beta cells." b. "Periods of stress cause increases in glycogen production by the adrenal cortex." c. "Stress is directly associated with decreased insulin tolerance." d. "The inhibition of beta cells to glucose is increased in periods of stress."

ANS: B "Periods of stress cause increases in glycogen production by the adrenal cortex." Stress stimulates the adrenal cortex to release glucocorticoids, which can cause hyperglycemia.

A nurse assesses a client with diabetes mellitus and notes the client only responds to a sternal rub by moaning, has capillary blood glucose of 33 g/dL, and has an intravenous line that is infiltrated with 0.45% normal saline. Which action should the nurse take first? a. Administer 1 mg of intramuscular glucagon. b. Encourage the client to drink orange juice. c. Insert a new intravenous access line. d. Administer 25 mL dextrose 50% (D50) IV push.

ANS: A a. Administer 1 mg of intramuscular glucagon. The clients blood glucose level is dangerously low. The nurse needs to administer glucagon IM immediately to increase the clients blood glucose level. The nurse should insert a new IV after administering the glucagon and can use the new IV site for future doses of D50 if the clients blood glucose level does not rise. Once the client is awake, orange juice may be administered orally along with a form of protein such as a peanut butter.

An emergency department nurse assesses a client with ketoacidosis. Which clinical manifestation should the nurse correlate with this condition? a. Increased rate and depth of respiration b. Extremity tremors followed by seizure activity c. Oral temperature of 102 F (38.9 C) d. Severe orthostatic hypotension

ANS: A a. Increased rate and depth of respiration Ketoacidosis decreases the pH of the blood, stimulating the respiratory control areas of the brain to buffer the effects of increasing acidosis. The rate and depth of respiration are increased (Kussmaul respirations) in an attempt to excrete more acids by exhalation. Tremors, elevated temperature, and orthostatic hypotension are not associated with ketoacidosis.

A nurse teaches a client who is diagnosed with diabetes mellitus. Which statement should the nurse include in this clients plan of care to delay the onset of microvascular and macrovascular complications? a. Maintain tight glycemic control and prevent hyperglycemia. b. Restrict your fluid intake to no more than 2 liters a day. c. Prevent hypoglycemia by eating a bedtime snack. d. Limit your intake of protein to prevent ketoacidosis.

ANS: A a. Maintain tight glycemic control and prevent hyperglycemia. Hyperglycemia is a critical factor in the pathogenesis of long-term diabetic complications. Maintaining tight glycemic control will help delay the onset of complications. Restricting fluid intake is not part of the treatment plan for clients with diabetes. Preventing hypoglycemia and ketosis, although important, are not as important as maintaining daily glycemic control.

After teaching a client with diabetes mellitus to inject insulin, the nurse assesses the clients understanding. Which statement made by the client indicates a need for additional teaching? a. The lower abdomen is the best location because it is closest to the pancreas. b. I can reach my thigh the best, so I will use the different areas of my thighs. c. By rotating the sites in one area, my chance of having a reaction is decreased. d. Changing injection sites from the thigh to the arm will change absorption rates.

ANS: A a. The lower abdomen is the best location because it is closest to the pancreas. The abdominal site has the fastest rate of absorption because of blood vessels in the area, not because of its proximity to the pancreas. The other statements are accurate assessments of insulin administration.

A nurse cares for a client with diabetes mellitus who is visually impaired. The client asks, Can I ask my niece to prefill my syringes and then store them for later use when I need them? How should the nurse respond? a. Yes. Prefilled syringes can be stored for 3 weeks in the refrigerator in a vertical position with the needle pointing up. b. Yes. Syringes can be filled with insulin and stored for a month in a location that is protected from light. c. Insulin reacts with plastic, so prefilled syringes are okay, but you will need to use glass syringes. d. No. Insulin syringes cannot be prefilled and stored for any length of time outside of the container.

ANS: A a. Yes. Prefilled syringes can be stored for 3 weeks in the refrigerator in a vertical position with the needle pointing up. Insulin is relatively stable when stored in a cool, dry place away from light. When refrigerated, prefilled plastic syringes are stable for up to 3 weeks. They should be stored in the refrigerator in the vertical position with the needle pointing up to prevent suspended insulin particles from clogging the needle.

A nurse cares for a client who has a family history of diabetes mellitus. The client states, My father has type 1 diabetes mellitus. Will I develop this disease as well? How should the nurse respond? a. Your risk of diabetes is higher than the general population, but it may not occur. b. No genetic risk is associated with the development of type 1 diabetes mellitus. c. The risk for becoming a diabetic is 50% because of how it is inherited. d. Female children do not inherit diabetes mellitus, but male children will.

ANS: A a. Your risk of diabetes is higher than the general population, but it may not occur. Risk for type 1 diabetes is determined by inheritance of genes coding for HLA-DR and HLA-DQ tissue types. Clients who have one parent with type 1 diabetes are at increased risk for its development. Diabetes (type 1) seems to require interaction between inherited risk and environmental factors, so not everyone with these genes develops diabetes. The other statements are not accurate.

A nurse teaches a client with diabetes mellitus about foot care. Which statements should the nurse include in this clients teaching? (Select all that apply.) a. Do not walk around barefoot. b. Soak your feet in a tub each evening. c. Trim toenails straight across with a nail clipper. d. Treat any blisters or sores with Epsom salts. e. Wash your feet every other day.

ANS: A, C a. Do not walk around barefoot. c. Trim toenails straight across with a nail clipper. Clients who have diabetes mellitus are at high risk for wounds on the feet secondary to peripheral neuropathy and poor arterial circulation. The client should be instructed to not walk around barefoot or wear sandals with open toes. These actions place the client at higher risk for skin breakdown of the feet. The client should be instructed to trim toenails straight across with a nail clipper. Feet should be washed daily with lukewarm water and soap, but feet should not be soaked in the tub. The client should contact the provider immediately if blisters or sores appear and should not use home remedies to treat these wounds.

Which statement(s) explain(s) a reason for weight loss in type 1 diabetics? (select all that apply.) a. Loss of body fluid b. Insulin intolerance c. Metabolization of body fats d. Stress of disease e. Altered diet

ANS: A, C a. Loss of body fluid c. Metabolization of body fats Weight loss in type 1 diabetics can be attributed to loss of body fluids and metabolization of fats. Insulin intolerance, stress of the disease, and altered diet are not reasons for weight loss in type 1 diabetes.

What are functional causes of hypoglycemia? (Select all that apply.) a. Dumping syndrome b. Overdose of insulin c. Addison disease d. Prolonged muscular exercise e. Chronic alcoholism

ANS: A, C, D Dumping syndrome, Addison disease, and prolonged exercise are functional causes of hypoglycemia. Overdose of insulin and chronic alcoholism are exogenous causes.

When discussing exercise programs with the diabetic, which instruction(s) is/are important for the nurse to include? (select all that apply.) a. Delay exercise until glucose controlled. b. Check glucose immediately after exercising. c. Keep a quick source of glucose readily available while exercising. d. Begin slowly and build up to 30 to 45 minutes. e. Only use the abdominal injection site for insulin.

ANS: A, C, D, E a. Delay exercise until glucose controlled. c. Keep a quick source of glucose readily available while exercising. d. Begin slowly and build up to 30 to 45 minutes. e. Only use the abdominal injection site for insulin. The patient should delay exercise until glucose is controlled, keep a quick source of glucose readily available, begin slowly and build, and use the abdominal injection site for insulin. The patient should check the glucose level before exercising.

A nurse assesses a client who is experiencing diabetic ketoacidosis (DKA). For which manifestations should the nurse monitor the client? (Select all that apply.) a. Deep and fast respirations b. Decreased urine output c. Tachycardia d. Dependent pulmonary crackles e. Orthostatic hypotension

ANS: A, C, E DKA leads to dehydration, which is manifested by tachycardia and orthostatic hypotension. Usually clients have Kussmaul respirations, which are fast and deep. Increased urinary output (polyuria) is severe. Because of diuresis and dehydration, peripheral edema and crackles do not occur.

What should a teaching plan about foot care include for a patient with diabetes? (Select all that apply.) a. Wash and carefully dry the feet every day. b. Apply lotion between the toes. c. Protect the feet from extreme temperatures. d. Walk barefoot only indoors. e. Buy shoes that are comfortable and supportive.

ANS: A, C, E Washing, inspecting, and drying the feet, especially between the toes, are essential. Protecting the feet from heat and cold and wearing supportive shoes are important to good foot health. Lotion can be applied to the soles and tops of the feet but not between the toes. Walking barefoot is contraindicated for a person with diabetes.

Which genetic factor(s) increase(s) the risk of a person developing diabetes mellitus (DM)? (select all that apply.) a. Number of relatives with DM b. Body mass index (BMI) c. Sedentary lifestyle d. Genetic closeness of relatives with DM e. Race

ANS: A, D, E a. Number of relatives with DM d. Genetic closeness of relatives with DM e. Race Genetic factors that increase the risk of developing diabetes include the number and genetic closeness of relatives with diabetes, as well as race. BMI and sedentary lifestyle are not genetic factors.

A nurse assesses clients at a health fair. Which clients should the nurse counsel to be tested for diabetes? (Select all that apply.) a. 56-year-old African-American male b. Female with a 30-pound weight gain during pregnancy c. Male with a history of pancreatic trauma d. 48-year-old woman with a sedentary lifestyle e. Male with a body mass index greater than 25 kg/m2 f. 28-year-old female who gave birth to a baby weighing 9.2 pounds

ANS: A, D, E, F Risk factors for type 2 diabetes include certain ethnic/racial groups (African Americans, American Indians, Hispanics), obesity and physical inactivity, and giving birth to large babies. Pancreatic trauma and a 30-pound gestational weight gain are not risk factors.

The nurse is caring for a patient with ketosis. Which statement indicates that the patient correctly understands the phenomenon? a. "I took too much insulin to decrease my body's glucose levels." b. "The condition resulted when my body tried to break down and use my stores of fats." c. "My blood glucose went over 150 mg/dL and caused this condition." d. "I exercised too much reduced my blood glucose level too dramatically."

ANS: B "The condition resulted when my body tried to break down and use my stores of fats." People with type 1 diabetes are more prone to a serious complication, ketosis, associated with an excess production of ketone bodies, leading to ketoacidosis (metabolic acidosis). When the glucose level gets too high, the body attempts to metabolize fats for energy, and the result is a buildup of ketone bodies.

The nurse is discussing insulin administration with an assigned patient. The patient reports that she prefers to use only certain sites for insulin injections and questions the need to rotate sites. What response by the nurse is most appropriate? a. "Rotating injection sites helps reduce your risk of infection." b. "Rotating injection sites helps enhance insulin absorption." c. "Unsightly fatty tumors can develop when you do not adequately rotate injection sites." d. "Rotating injection sites decreases your risk of an insulin reaction."

ANS: B "Rotating injection sites helps enhance insulin absorption." Insulin injections are rotated within one body area to enhance absorption. Patients are given charts showing the places on the arms, legs, buttocks, and abdomen where insulin can be injected. Patients should be encouraged to keep a daily record of injection sites to help remember which sites have been used and to avoid the problem of altered or erratic absorption, which is a complication associated with overuse of a single site. The most important way to reduce the incidence of infection is to wash the hands before insulin administration and to avoid reusing syringes. Fatty tumors are not complications of overuse of a single injection site. The term insulin reaction refers to hypoglycemia, and hypoglycemia is not directly associated with the failure to rotate injection sites.

The patient comes to the emergency room complaining of abdominal pain. The nurse assesses dry, hot skin, fruity breath, and deep respirations. To which problem should the nurse attribute these findings? a. An insulin reaction b. Ketoacidosis c. Rebound hyperglycemia d. Hypoglycemia

ANS: B An insulin reaction Abdominal pain with dry, hot skin, fruity breath, and deep respirations is characteristic of ketoacidosis. Manifestations of an insulin reaction, or hypoglycemia, include tremulousness, hunger, headache, pallor, sweating, palpitations, blurred vision, and weakness. Rebound hyperglycemia, or the Somogyi effect, follows a period of hypoglycemia, often during sleep. When hypoglycemia occurs, the body secretes glucagon, epinephrine, growth hormone, and cortisol to counteract the effects of low blood sugar. The patient may report nightmares and night sweats along with morning elevated serum glucose; if the patient increases the insulin dose, it worsens the problem.

Which reason best explains why diabetics are prone to infection? a. High glucose levels provide an environment conducive to bacterial growth. b. Atherosclerotic vascular changes decrease blood supply to tissues. c. Diabetics display abnormal phagocyte function. d. Diabetics display decreased leukocyte function.

ANS: B Atherosclerotic vascular changes decrease blood supply to tissues. The primary reason for increased risk of infection in diabetic patients is the hyperglycemic environment. Lesser risk factors include atherosclerotic vascular changes, abnormal phagocyte function, and decreased leukocyte function.

A patient recently diagnosed with type 1 diabetes mellitus (DM) asks why she is experiencing increased thirst. Which explanation is most appropriate? a. Diabetes results in a lack of protein absorption that decreases amino acids and causes increased thirst. b. High glucose levels in the blood pull cellular water into circulating volume and increase thirst. c. Thirst results from the body's increased loss of fluids from frequent urination. d. Diabetes causes large amount of fluid to shut to the pancreas, which dehydrates the body.

ANS: B High glucose levels in the blood pull cellular water into circulating volume and increase thirst. Polydipsia is stimulated by cellular dehydration from the hyperglycemia pulling intracellular fluid into the circulating volume.

A nurse cares for a client with diabetes mellitus who asks, Why do I need to administer more than one injection of insulin each day? How should the nurse respond? a. You need to start with multiple injections until you become more proficient at self-injection. b. A single dose of insulin each day would not match your blood insulin levels and your food intake patterns. c. A regimen of a single dose of insulin injected each day would require that you eat fewer carbohydrates. d. A single dose of insulin would be too large to be absorbed, predictably putting you at risk for insulin shock.

ANS: B b. A single dose of insulin each day would not match your blood insulin levels and your food intake patterns. Even when a single injection of insulin contains a combined dose of different-acting insulin types, the timing of the actions and the timing of food intake may not match well enough to prevent wide variations in blood glucose levels. One dose of insulin would not be appropriate even if the client decreased carbohydrate intake. Additional injections are not required to allow the client practice with injections, nor will one dose increase the clients risk of insulin shock.

A nurse teaches a client with type 2 diabetes mellitus who is prescribed glipizide (Glucotrol). Which statement should the nurse include in this clients teaching? a. Change positions slowly when you get out of bed. b. Avoid taking nonsteroidal anti-inflammatory drugs (NSAIDs). c. If you miss a dose of this drug, you can double the next dose. d. Discontinue the medication if you develop a urinary infection.

ANS: B b. Avoid taking nonsteroidal anti-inflammatory drugs (NSAIDs). NSAIDs potentiate the hypoglycemic effects of sulfonylurea agents. Glipizide is a sulfonylurea. The other statements are not applicable to glipizide.

A nurse teaches a client about self-monitoring of blood glucose levels. Which statement should the nurse include in this clients teaching to prevent blood borne infections? a. Wash your hands after completing each test. b. Do not share your monitoring equipment. c. Blot excess blood from the strip with a cotton ball. d. Use gloves when monitoring your blood glucose.

ANS: B b. Do not share your monitoring equipment. Small particles of blood can adhere to the monitoring device, and infection can be transported from one user to another. Hepatitis B in particular can survive in a dried state for about a week. The client should be taught to avoid sharing any equipment, including the lancet holder. The client should be taught to wash his or her hands before testing. The client would not need to blot excess blood away from the strip or wear gloves.

A nurse cares for a client who is prescribed pioglitazone (Actos). After 6 months of therapy, the client reports that his urine has become darker since starting the medication. Which action should the nurse take? a. Assess for pain or burning with urination. b. Review the clients liver function study results. c. Instruct the client to increase water intake. d. Test a sample of urine for occult blood.

ANS: B b. Review the clients liver function study results. Thiazolidinediones (including pioglitazone) can affect liver function; liver function should be assessed at the start of therapy and at regular intervals while the client continues to take these drugs. Dark urine is one indicator of liver impairment because bilirubin is increased in the blood and is excreted in the urine. The nurse should check the clients most recent liver function studies. The nurse does not need to assess for pain or burning with urination and does not need to check the urine for occult blood. The client does not need to be told to increase water intake.

A nurse assesses a client who has diabetes mellitus. Which arterial blood gas values should the nurse identify as potential ketoacidosis in this client? a. pH 7.38, HCO3 22 mEq/L, PCO2 38 mm Hg, PO2 98 mm Hg b. pH 7.28, HCO3 18 mEq/L, PCO2 28 mm Hg, PO2 98 mm Hg c. pH 7.48, HCO3 28 mEq/L, PCO2 38 mm Hg, PO2 98 mm Hg d. pH 7.32, HCO3 22 mEq/L, PCO2 58 mm Hg, PO2 88 mm Hg

ANS: B b. pH 7.28, HCO3 18 mEq/L, PCO2 28 mm Hg, PO2 98 mm Hg When the lungs can no longer offset acidosis, the pH decreases to below normal. A client who has diabetic ketoacidosis would present with arterial blood gas values that show primary metabolic acidosis with decreased bicarbonate levels and a compensatory respiratory alkalosis with decreased carbon dioxide levels.

A nurse is teaching a client with diabetes mellitus who asks, Why is it necessary to maintain my blood glucose levels no lower than about 60 mg/dL? How should the nurse respond? a. Glucose is the only fuel used by the body to produce the energy that it needs. b. Your brain needs a constant supply of glucose because it cannot store it. c. Without a minimum level of glucose, your body does not make red blood cells. d. Glucose in the blood prevents the formation of lactic acid and prevents acidosis.

ANS: B Because the brain cannot synthesize or store significant amounts of glucose, a continuous supply from the bodys circulation is needed to meet the fuel demands of the central nervous system. The nurse would want to educate the client to prevent hypoglycemia. The body can use other sources of fuel, including fat and protein, and glucose is not involved in the production of red blood cells. Glucose in the blood will encourage glucose metabolism but is not directly responsible for lactic acid formation.

Which requirement(s) is/are part of the criteria for "tight control" of hyperglycemia? (select all that apply.) a. Perform glucose testing twice daily. b. Administer insulin injections three times a day based on glucometer readings. c. Maintain fasting glucose within normal limits. d. Maintain normal weight for height and age. e. Maintain cholesterol within normal limits.

ANS: B, C, D, E b. Administer insulin injections three times a day based on glucometer readings. c. Maintain fasting glucose within normal limits. d. Maintain normal weight for height and age. e. Maintain cholesterol within normal limits. Patients attempting tight control follow an intensive therapy plan of blood glucose testing and insulin injections, three or more times a day, or they use an insulin pump. Maintaining a normal fasting glucose, weight for height and age, and cholesterol helps establish "tight control" of hyperglycemia.

The nurse is educating a patient with gestational diabetes. Which statement indicates that the patient needs additional teaching? a. "Gestational diabetes happens because of the hormonal changes of pregnancy." b. "I should exercise regularly and lose weight to reduce my risk of becoming a diabetic." c. "This problem goes away completely once I give birth." d. "The baby will have to be monitored for hypoglycemia during my pregnancy."

ANS: C "This problem goes away completely once I give birth." Giving birth does not automatically resolve gestational diabetes. Of the women who have gestational diabetes, 5% to 10% go on to develop type 2 diabetes. The patient correctly understands that gestational diabetes occurs because of hormonal changes in pregnancy, proper diet and regular exercise may help decrease the likelihood of developing type 2 diabetes, and the baby will require monitoring for hypoglycemia throughout the patient's pregnancy.

The patient takes his NovoLog 70/30 at 0700. When should the nurse suggest that the patient schedule exercise? a. 0730. b. 1000. c. 1300. d. Scheduling exercise is unnecessary.

ANS: C 1300 Exercise should occur after peak action time to prevent hypoglycemia. NovoLog is a rapid-acting insulin that peaks 1 to 3 hours after administration. Since the insulin is administered at 70/30, scheduling exercise for 1300 would mean that it occurs after the peak insulin action.

The nurse is caring for a patient with type 1 diabetes who is diaphoretic and clammy. The patient complains of hunger but denies pain. The nurse performs a bedside blood glucose check. What should the nurse do next? a. Administer insulin as scheduled. b. Notify the charge nurse. c. Give 6 ounces of orange juice. d. Document the findings.

ANS: C Give 6 ounces of orange juice. These findings are consistent with hypoglycemia; manifestations of hypoglycemia include tremulousness, hunger, headache, palpitations, blurred vision, and weakness. Management includes providing a source of quick-acting carbohydrate/glucose such as orange juice. The nurse should withhold the patient's scheduled insulin at this time. The nurse should document the findings and then notify the charge nurse.

The nurse is caring for an older adult patient who is diabetic. The nurse cautions against the technique of "tight control" of hyperglycemia. Which statement explains why this management method is not recommended? a. Older adults may not accurately test and administer sliding-scale insulin. b. Older adults possess lower risk for hyperglycemia. c. Older adults may experience cardiovascular problems from hypoglycemia. d. Older adults possess an unstable metabolic rate.

ANS: C Older adults may experience cardiovascular problems from hypoglycemia. One complication of the "tight control" method includes hypoglycemia. Older adults experience hypoglycemia more quickly than do younger people, and older adults are more prone to hypoglycemic episodes. The older adult may progress to dangerously low levels of blood glucose before signs and symptoms are obvious. Severe hypoglycemia in the older adult can precipitate myocardial infarction, angina, stroke, or seizures. For this reason, "tight control" may not be the best thing for the older adult. Older adults can accurately test and administer insulin, possess a higher risk for hypoglycemia, and do not possess an unstable metabolic rate.

A nurse assesses a client who has a 15-year history of diabetes and notes decreased tactile sensation in both feet. Which action should the nurse take first? a. Document the finding in the clients chart. b. Assess tactile sensation in the clients hands. c. Examine the clients feet for signs of injury. d. Notify the health care provider.

ANS: C c. Examine the clients feet for signs of injury. Diabetic neuropathy is common when the disease is of long duration. The client is at great risk for injury in any area with decreased sensation because he or she is less able to feel injurious events. Feet are common locations for neuropathy and injury, so the nurse should inspect them for any signs of injury. After assessment, the nurse should document findings in the clients chart. Testing sensory perception in the hands may or may not be needed. The health care provider can be notified after assessment and documentation have been completed.

The nurse watches a patient perform an insulin injection. Which observation(s) indicate(s) that the patient needs additional instruction? (select all that apply.) a. The patient uses a 90-degree angle to administer the injection. b. The patient cleans the injection site with alcohol before the injection. c. The patient rubs the injection site after administration of the insulin injection. d. The patient draws up the cloudy insulin and then the clear insulin. e. The patient shakes the insulin bottle before administration.

ANS: C, D, E c. The patient rubs the injection site after administration of the insulin injection. d. The patient draws up the cloudy insulin and then the clear insulin. e. The patient shakes the insulin bottle before administration. The patient should not rub the injection site because it could alter absorption. When mixing two types of insulin, in order to prevent contamination of the second vial, the patient should withdraw clear insulin into the syringe first. Shaking the bottle can damage the solution; the patient should gently roll the bottle between the palms of the hands. Administering the injection at a 90-degree angle and cleaning the injection site prior to injection describe appropriate technique.

Type 2 diabetes cases compose approximately what percentage of all known cases of diabetes? a. 70% b. 75% c. 80% d. 95%

ANS: D 95% Type 2 diabetics comprise 90% to 95% of all known cases.

A patient with type 1 diabetes mellitus (DM) is preparing for a moderate 30-minute exercise period. Which action best indicates that the patient understands condition management? a. The patient reduces insulin use during days when exercise periods are planned. b. The patient administers insulin after exercise rather than before exercise. c. The patient eats a high-carbohydrate snack before the exercise period. d. The patient consumes a simple carbohydrate snack after 30 minutes of activity.

ANS: D The patient consumes a simple carbohydrate snack after 30 minutes of activity. During moderate exercise (such as brisk walking, bowling, or vacuuming), 5 g of simple carbohydrate should be consumed at the end of 30 minutes and at 30-minute intervals during the continued activity. (A food example with 5 g of simple carbohydrate is 1 tsp honey.)

After teaching a client who has diabetes mellitus and proliferative retinopathy, nephropathy, and peripheral neuropathy, the nurse assesses the clients understanding. Which statement made by the client indicates a correct understanding of the teaching? a. I have so many complications; exercising is not recommended. b. I will exercise more frequently because I have so many complications. c. I used to run for exercise; I will start training for a marathon. d. I should look into swimming or water aerobics to get my exercise.

ANS: D d. I should look into swimming or water aerobics to get my exercise. Exercise is not contraindicated for this client, although modifications based on existing pathology are necessary to prevent further injury. Swimming or water aerobics will give the client exercise without the worry of having the correct shoes or developing a foot injury. The client should not exercise too vigorously.

After teaching a client with type 2 diabetes mellitus who is prescribed nateglinide (Starlix), the nurse assesses the clients understanding. Which statement made by the client indicates a correct understanding of the prescribed therapy? a. Ill take this medicine during each of my meals. b. I must take this medicine in the morning when I wake. c. I will take this medicine before I go to bed. d. I will take this medicine immediately before I eat.

ANS: D d. I will take this medicine immediately before I eat. Nateglinide is an insulin secretagogue that is designed to increase meal-related insulin secretion. It should be taken immediately before each meal. The medication should not be taken without eating as it will decrease the clients blood glucose levels. The medication should be taken before meals instead of during meals.

A nurse reviews laboratory results for a client with diabetes mellitus who presents with polyuria, lethargy, and a blood glucose of 560 mg/dL. Which laboratory result should the nurse correlate with the clients polyuria? a. Serum sodium: 163 mEq/L b. Serum creatinine: 1.6 mg/dL c. Presence of urine ketone bodies d. Serum osmolarity: 375 mOsm/kg

ANS: D Hyperglycemia causes hyperosmolarity of extracellular fluid. This leads to polyuria from an osmotic diuresis. The clients serum osmolarity is high. The clients sodium would be expected to be high owing to dehydration. Serum creatinine and urine ketone bodies are not related to the polyuria.

A patient has been brought to the emergency department by paramedics after being found unconscious. The patients Medic Alert bracelet indicates that the patient has type 1 diabetes and the patients blood glucose is 22 mg/dL (1.2 mmol/L). The nurse should anticipate what intervention? A) IV administration of 50% dextrose in water B) Subcutaneous administration of 10 units of Humalog C) Subcutaneous administration of 12 to 15 units of regular insulin D) IV bolus of 5% dextrose in 0.45% NaCl

Ans: A IV administration of 50% dextrose in water Feedback: In hospitals and emergency departments, for patients who are unconscious or cannot swallow, 25 to 50 mL of 50% dextrose in water (D50W) may be administered IV for the treatment of hypoglycemia. Five percent dextrose would be inadequate and insulin would exacerbate the patients condition.

A nurse is conducting a class on how to self-manage insulin regimens. A patient asks how long a vial of insulin can be stored at room temperature before it goes bad. What would be the nurses best answer? A) If you are going to use up the vial within 1 month it can be kept at room temperature. B) If a vial of insulin will be used up within 21 days, it may be kept at room temperature. C) If a vial of insulin will be used up within 2 weeks, it may be kept at room temperature. D) If a vial of insulin will be used up within 1 week, it may be kept at room temperature.

Ans: A If you are going to use up the vial within 1 month it can be kept at room temperature. Feedback: If a vial of insulin will be used up within 1 month, it may be kept at room temperature.

A diabetes nurse educator is teaching a group of patients with type 1 diabetes about sick day rules. What guideline applies to periods of illness in a diabetic patient? A) Do not eliminate insulin when nauseated and vomiting. B) Report elevated glucose levels greater than 150 mg/dL. C) Eat three substantial meals a day, if possible. D) Reduce food intake and insulin doses in times of illness.

Ans: A A) Do not eliminate insulin when nauseated and vomiting. Feedback: The most important issue to teach patients with diabetes who become ill is not to eliminate insulin doses when nausea and vomiting occur. Rather, they should take their usual insulin or oral hypoglycemic agent dose, then attempt to consume frequent, small portions of carbohydrates. In general, blood sugar levels will rise but should be reported if they are greater than 300 mg/dL.

A diabetic nurse is working for the summer at a camp for adolescents with diabetes. When providing information on the prevention and management of hypoglycemia, what action should the nurse promote? A) Always carry a form of fast-acting sugar. B) Perform exercise prior to eating whenever possible. C) Eat a meal or snack every 8 hours. D) Check blood sugar at least every 24 hours.

Ans: A Always carry a form of fast-acting sugar. Feedback: The following teaching points should be included in information provided to the patient on how to prevent hypoglycemia: Always carry a form of fast-acting sugar, increase food prior to exercise, eat a meal or snack every 4 to 5 hours, and check blood sugar regularly.

A patient presents to the clinic complaining of symptoms that suggest diabetes. What criteria would support checking blood levels for the diagnosis of diabetes? a. Fasting plasma glucose greater than or equal to 126 mg/dL b. Random plasma glucose greater than 150 mg/dL c. Fasting plasma glucose greater than 116 mg/dL on 2 separate occasions d. Random plasma glucose greater than 126 mg/dL

Ans: A Fasting plasma glucose greater than or equal to 126 mg/dL Feedback: Criteria for the diagnosis of diabetes include symptoms of diabetes plus random plasma glucose greater than or equal to 200 mg/dL, or a fasting plasma glucose greater than or equal to 126 mg/dL.

A diabetic educator is discussing sick day rules with a newly diagnosed type 1 diabetic. The educator is aware that the patient will require further teaching when the patient states what? a. I will not take my insulin on the days when I am sick, but I will certainly check my blood sugar every 2 hours. b. If I cannot eat a meal, I will eat a soft food such as soup, gelatin, or pudding six to eight times a day. c. I will call the doctor if I am not able to keep liquids in my body due to vomiting or diarrhea. d. I will call the doctor if my blood sugar is over 300 mg/dL or if I have ketones in my urine

Ans: A I will not take my insulin on the days when I am sick, but I will certainly check my blood sugar every 2 hours. Feedback: The nurse must explanation the sick day rules again to the patient who plans to stop taking insulin when sick. The nurse should emphasize that the patient should take insulin agents as usual and test ones blood sugar and urine ketones every 3 to 4 hours. In fact, insulin-requiring patients may need supplemental doses of regular insulin every 3 to 4 hours. The patient should report elevated glucose levels (greater than 300 mg/dL or as otherwise instructed) or urine ketones to the physician. If the patient is not able to eat normally, the patient should be instructed to substitute soft foods such a gelatin, soup, and pudding. If vomiting, diarrhea, or fever persists, the patient should have an intake of liquids every 30 to 60 minutes to prevent dehydration.

A patient has been living with type 2 diabetes for several years, and the nurse realizes that the patient is likely to have minimal contact with the health care system. In order to ensure that the patient maintains adequate blood sugar control over the long term, the nurse should recommend which of the following? A) Participation in a support group for persons with diabetes B) Regular consultation of websites that address diabetes management C) Weekly telephone check-ins with an endocrinologist D) Participation in clinical trials relating to antihyperglycemics

Ans: A Participation in a support group for persons with diabetes Feedback: Participation in support groups is encouraged for patients who have had diabetes for many years as well as for those who are newly diagnosed. This is more interactive and instructive than simply consulting websites. Weekly telephone contact with an endocrinologist is not realistic in most cases. Participation in research trials may or may not be beneficial and appropriate, depending on patients circumstances.

The most recent blood work of a patient with a longstanding diagnosis of type 1 diabetes has shown the presence of microalbuminuria. What is the nurses most appropriate action? a. Teach the patient about actions to slow the progression of nephropathy. b. Ensure that the patient receives a comprehensive assessment of liver function. c. Determine whether the patient has been using expired insulin. d. Administer a fluid challenge and have the test repeated.

Ans: A Teach the patient about actions to slow the progression of nephropathy. Feedback: Clinical nephropathy eventually develops in more than 85% of people with microalbuminuria. As such, educational interventions addressing this microvascular complication are warranted. Expired insulin does not cause nephropathy, and the patients liver function is not likely affected. There is no indication for the use of a fluid challenge.

A 15-year-old child is brought to the emergency department with symptoms of hyperglycemia and is subsequently diagnosed with diabetes. Based on the fact that the childs pancreatic beta cells are being destroyed, the patient would be diagnosed with what type of diabetes? A) Type 1 diabetes B) Type 2 diabetes C) Noninsulin-dependent diabetes D) Prediabetes

Ans: A Type 1 diabetes Feedback: Beta cell destruction is the hallmark of type 1 diabetes. Noninsulin-dependent diabetes is synonymous with type 2 diabetes, which involves insulin resistance and impaired insulin secretion, but not beta cell destruction. Prediabetes is characterized by normal glucose metabolism, but a previous history of hyperglycemia, often during illness or pregnancy.

A patient with a longstanding diagnosis of type 1 diabetes has a history of poor glycemic control. The nurse recognizes the need to assess the patient for signs and symptoms of peripheral neuropathy. Peripheral neuropathy constitutes a risk for what nursing diagnosis? A) Infection B) Acute pain C) Acute confusion D) Impaired urinary elimination

Ans: A infection Feedback: Decreased sensations of pain and temperature place patients with neuropathy at increased risk for injury and undetected foot infections. The neurologic changes associated with peripheral neuropathy do not normally result in pain, confusion, or impairments in urinary function.

A nurse is caring for a patient with type 1 diabetes who is being discharged home tomorrow. What is the best way to assess the patients ability to prepare and self-administer insulin? A) Ask the patient to describe the process in detail. B) Observe the patient drawing up and administering the insulin. C) Provide a health education session reviewing the main points of insulin delivery. D) Review the patients first hemoglobin A1C result after discharge.

Ans: B Observe the patient drawing up and administering the insulin. Feedback: Nurses should assess the patients ability to perform diabetes related self-care as soon as possible during the hospitalization or office visit to determine whether the patient requires further diabetes teaching. While consulting a home care nurse is beneficial, an initial assessment should be performed during the hospitalization or office visit. Nurses should directly observe the patient performing the skills such as insulin preparation and infection, blood glucose monitoring, and foot care. Simply questioning the patient about these skills without actually observing performance of the skill is not sufficient. Further education does not guarantee learning.

A medical nurse is caring for a patient with type 1 diabetes. The patients medication administration record includes the administration of regular insulin three times daily. Knowing that the patients lunch tray will arrive at 11:45, when should the nurse administer the patients insulin? a. 10:45 b. 11:15 c. 11:45 d. 11:50

Ans: B 11:15 Feedback: Regular insulin is usually administered 2030 min before a meal. Earlier administration creates a risk for hypoglycemia; later administration creates a risk for hyperglycemia.

A patient has just been diagnosed with type 2 diabetes. The physician has prescribed an oral antidiabetic agent that will inhibit the production of glucose by the liver and thereby aid in the control of blood glucose. What type of oral antidiabetic agent did the physician prescribe for this patient? A) A sulfonylurea B) A biguanide C) A thiazolidinedione D) An alpha glucosidase inhibitor

Ans: B A biguanide Feedback: Sulfonylureas exert their primary action by directly stimulating the pancreas to secrete insulin and therefore require a functioning pancreas to be effective. Biguanides inhibit the production of glucose by the liver and are in used in type 2 diabetes to control blood glucose levels. Thiazolidinediones enhance insulin action at the receptor site without increasing insulin secretion from the beta cells of the pancreas. Alpha glucosidase inhibitors work by delaying the absorption of glucose in the intestinal system, resulting in a lower postprandial blood glucose level.

A diabetes nurse is assessing a patients knowledge of self-care skills. What would be the most appropriate way for the educator to assess the patients knowledge of nutritional therapy in diabetes? A) Ask the patient to describe an optimally healthy meal. B) Ask the patient to keep a food diary and review it with the nurse. C)Ask the patients family what he typically eats. D) Ask the patient to describe a typical days food intake.

Ans: B Ask the patient to keep a food diary and review it with the nurse. Feedback: Reviewing the patients actual food intake is the most accurate method of gauging the patients diet.

An elderly patient comes to the clinic with her daughter. The patient is a diabetic and is concerned about foot care. The nurse goes over foot care with the patient and her daughter as the nurse realizes that foot care is extremely important. Why would the nurse feel that foot care is so important to this patient? a. An elderly patient with foot ulcers experiences severe foot pain due to the diabetic polyneuropathy. b. Avoiding foot ulcers may mean the difference between institutionalization and continued independent living. c. Hypoglycemia is linked with a risk for falls; this risk is elevated in older adults with diabetes. d. Oral antihyperglycemics have the possible adverse effect of decreased circulation to the lower extremities.

Ans: B Avoiding foot ulcers may mean the difference between institutionalization and continued independent living. Feedback: The nurse recognizes that providing information on the long-term complicationsespecially foot and eye problemsassociated with diabetes is important. Avoiding amputation through early detection of foot ulcers may mean the difference between institutionalization and continued independent living for the elderly person with diabetes. While the nurse recognizes that hypoglycemia is a dangerous situation and may lead to falls, hypoglycemia is not directly connected to the importance of foot care. Decrease in circulation is related to vascular changes and is not associated with drugs administered for diabetes.

An occupational health nurse is screening a group of workers for diabetes. What statement should the nurse interpret as suggestive of diabetes? A) Ive always been a fan of sweet foods, but lately Im turned off by them. B) Lately, I drink and drink and cant seem to quench my thirst. C) No matter how much sleep I get, it seems to take me hours to wake up. D) When I went to the washroom the last few days, my urine smelled odd.

Ans: B Lately, I drink and drink and cant seem to quench my thirst. Feedback: Classic clinical manifestations of diabetes include the three Ps: polyuria, polydipsia, and polyphagia. Lack of interest in sweet foods, fatigue, and foul-smelling urine are not suggestive of diabetes.

A patient with a history of type 1 diabetes has just been admitted to the critical care unit (CCU) for diabetic ketoacidosis. The CCU nurse should prioritize what assessment during the patients initial phase of treatment? A) Monitoring the patient for dysrhythmias B) Maintaining and monitoring the patients fluid balance C) Assessing the patients level of consciousness D) Assessing the patient for signs and symptoms of venous thromboembolism

Ans: B Maintaining and monitoring the patients fluid balance Feedback: In addition to treating hyperglycemia, management of DKA is aimed at correcting dehydration, electrolyte loss, and acidosis before correcting the hyperglycemia with insulin. The nurse should monitor the patient for dysrhythmias, decreased LOC and VTE, but restoration and maintenance of fluid balance is the highest priority.

A patient with type 2 diabetes achieves adequate glycemic control through diet and exercise. Upon being admitted to the hospital for a cholecystectomy, however, the patient has required insulin injections on two occasions. The nurse would identify what likely cause for this short-term change in treatment? A) Alterations in bile metabolism and release have likely caused hyperglycemia. B) Stress has likely caused an increase in the patients blood sugar levels. C) The patient has likely overestimated her ability to control her diabetes using nonpharmacologic measures. D) The patients volatile fluid balance surrounding surgery has likely caused unstable blood sugars.

Ans: B Stress has likely caused an increase in the patients blood sugar levels. Feedback: During periods of physiologic stress, such as surgery, blood glucose levels tend to increase, because levels of stress hormones (epinephrine, norepinephrine, glucagon, cortisol, and growth hormone) increase. The patients need for insulin is unrelated to the action of bile, the patients overestimation of previous blood sugar control, or fluid imbalance.

The nurse is discussing macrovascular complications of diabetes with a patient. The nurse would address what topic during this dialogue? a. The need for frequent eye examinations for patients with diabetes b. The fact that patients with diabetes have an elevated risk of myocardial infarction c. The relationship between kidney function and blood glucose levels d. The need to monitor urine for the presence of albumin

Ans: B The fact that patients with diabetes have an elevated risk of myocardial infarction Feedback: Myocardial infarction and stroke are considered macrovascular complications of diabetes, while the effects on vision and renal function are considered to be microvascular.

A diabetic patient calls the clinic complaining of having a flu bug. The nurse tells him to take his regular dose of insulin. What else should the nurse tell the patient? A) Make sure to stick to your normal diet. B) Try to eat small amounts of carbs, if possible. C) Ensure that you check your blood glucose every hour. D) For now, check your urine for ketones every 8 hours.

Ans: B Try to eat small amounts of carbs, if possible. Feedback: For prevention of DKA related to illness, the patient should attempt to consume frequent small portions of carbohydrates (including foods usually avoided, such as juices, regular sodas, and gelatin). Drinking fluids every hour is important to prevent dehydration. Blood glucose and urine ketones must be assessed every 3 to 4 hours.

40. A patient is brought to the emergency department by the paramedics. The patient is a type 2 diabetic and is experiencing HHS. The nurse should identify what components of HHS? Select all that apply. A) Leukocytosis B) Glycosuria C) Dehydration D) Hypernatremia E) Hyperglycemia

Ans: B, C, D, E B) Glycosuria C) Dehydration D) Hypernatremia E) Hyperglycemia Feedback: In HHS, persistent hyperglycemia causes osmotic diuresis, which results in losses of water and electrolytes. To maintain osmotic equilibrium, water shifts from the intracellular fluid space to the extracellular fluid space. With glycosuria and dehydration, hypernatremia and increased osmolarity occur. Leukocytosis does not take place.

A 28-year-old pregnant woman is spilling sugar in her urine. The physician orders a glucose tolerance test, which reveals gestational diabetes. The patient is shocked by the diagnosis, stating that she is conscientious about her health, and asks the nurse what causes gestational diabetes. The nurse should explain that gestational diabetes is a result of what etiologic factor? a. Increased caloric intake during the first trimester b. Changes in osmolality and fluid balance c. The effects of hormonal changes during pregnancy d. Over consumption of carbohydrates during the first two trimesters

Ans: C The effects of hormonal changes during pregnancy Feedback: Hyperglycemia and eventual gestational diabetes develops during pregnancy because of the secretion of placental hormones, which causes insulin resistance. The disease is not the result of food intake or changes in osmolality.

A diabetes educator is teaching a patient about type 2 diabetes. The educator recognizes that the patient understands the primary treatment for type 2 diabetes when the patient states what? A) I read that a pancreas transplant will provide a cure for my diabetes. B) I will take my oral antidiabetic agents when my morning blood sugar is high. C) I will make sure to follow the weight loss plan designed by the dietitian. D) I will make sure I call the diabetes educator when I have questions about my insulin.

Ans: C I will make sure to follow the weight loss plan designed by the dietitian. Feedback: Insulin resistance is associated with obesity; thus the primary treatment of type 2 diabetes is weight loss. Oral antidiabetic agents may be added if diet and exercise are not successful in controlling blood glucose levels. If maximum doses of a single category of oral agents fail to reduce glucose levels to satisfactory levels, additional oral agents may be used. Some patients may require insulin on an ongoing basis or on a temporary basis during times of acute psychological stress, but it is not the central component of type 2 treatment. Pancreas transplantation is associated with type 1 diabetes.

Which of the following patients with type 1 diabetes is most likely to experience adequate glucose control? a. A patient who skips breakfast when his glucose reading is greater than 220 mg/dL b. A patient who never deviates from her prescribed dose of insulin c. A patient who adheres closely to a meal plan and meal schedule d. A patient who eliminates carbohydrates from his daily intake

Ans: C A patient who adheres closely to a meal plan and meal schedule Feedback: The therapeutic goal for diabetes management is to achieve normal blood glucose levels without hypoglycemia. Therefore, diabetes management involves constant assessment and modification of the treatment plan by health professionals and daily adjustments in therapy (possibly including insulin) by patients. For patients who require insulin to help control blood glucose levels, maintaining consistency in the amount of calories and carbohydrates ingested at meals is essential. In addition, consistency in the approximate time intervals between meals, and the snacks, help maintain overall glucose control. Skipping meals is never advisable for person with type 1 diabetes.

A patient with type 1 diabetes mellitus is seeing the nurse to review foot care. What would be a priority instruction for the nurse to give the patient? A) Examine feet weekly for redness, blisters, and abrasions. B) Avoid the use of moisturizing lotions. C) Avoid hot-water bottles and heating pads. D) Dry feet vigorously after each bath.

Ans: C Avoid hot-water bottles and heating pads. Feedback: High-risk behaviors, such as walking barefoot, using heating pads on the feet, wearing open-toed shoes, soaking the feet, and shaving calluses, should be avoided. Socks should be worn for warmth. Feet should be examined each day for cuts, blisters, swelling,redness, tenderness, and abrasions. Lotion should be applied to dry feet but never between the toes. After a bath, the patient should gently, not vigorously, pat feet dry to avoid injury.

A nurse is providing health education to an adolescent newly diagnosed with type 1 diabetes mellitus and her family. The nurse teaches the patient and family that which of the following nonpharmacologic measures will decrease the bodys need for insulin? a. Adequate sleep b. Low stimulation c. Exercise d. Low-fat diet

Ans: C Exercise Feedback: Exercise lowers blood glucose, increases levels of HDLs, and decreases total cholesterol and triglyceride levels. Low fat intake and low levels of stimulation do not reduce a patients need for insulin. Adequate sleep is beneficial in reducing stress, but does not have an effect that is pronounced as that of exercise.

A student with diabetes tells the school nurse that he is feeling nervous and hungry. The nurse assesses the child and finds he has tachycardia and is diaphoretic with a blood glucose level of 50 mg/dL (2.8 mmol/L). What should the school nurse administer? A) A combination of protein and carbohydrates, such as a small cup of yogurt B) Two teaspoons of sugar dissolved in a cup of apple juice C) Half of a cup of juice, followed by cheese and crackers D) Half a sandwich with a protein-based filling

Ans: C Half of a cup of juice, followed by cheese and crackers Feedback: Initial treatment for hypoglycemia is 15 g concentrated carbohydrate, such as two or three glucose tablets, 1 tube glucose gel, or 0.5 cup juice. After initial treatment, the nurse should follow with a snack including starch and protein, such as cheese and crackers, milk and crackers, or half of a sandwich. It is unnecessary to add sugar to juice, even it if is labeled as unsweetened juice, because the fruit sugar in juice contains enough simple carbohydrate to raise the blood glucose level and additional sugar may result in a sharp rise in blood sugar that will last for several hours.

A patient newly diagnosed with type 2 diabetes is attending a nutrition class. What general guideline would be important to teach the patients at this class? a. Low fat generally indicates low sugar. b. Protein should constitute 30% to 40% of caloric intake. c. Most calories should be derived from carbohydrates. d. Animal fats should be eliminated from the diet.

Ans: C Most calories should be derived from carbohydrates. Feedback: Currently, the ADA and the Academy of Nutrition and Dietetics (formerly the American Dietetic Association) recommend that for all levels of caloric intake, 50% to 60% of calories should be derived from carbohydrates, 20% to 30% from fat, and the remaining 10% to 20% from protein.Low fat does not automatically mean low sugar. Dietary animal fat does not need to be eliminated from the diet.

24. A physician has explained to a patient that he has developed diabetic neuropathy in his right foot. Later that day, the patient asks the nurse what causes diabetic neuropathy. What would be the nurses best response? A) Research has shown that diabetic neuropathy is caused by fluctuations in blood sugar that have gone on for years. B) The cause is not known for sure but it is thought to have something to do with ketoacidosis. C) The cause is not known for sure but it is thought to involve elevated blood glucose levels over a period of years. D) Research has shown that diabetic neuropathy is caused by a combination of elevated glucose levels and elevated ketone levels.

Ans: C The cause is not known for sure but it is thought to involve elevated blood glucose levels over a period of years. Feedback The etiology of neuropathy may involve elevated blood glucose levels over a period of years. High blood sugars (rather than fluctuations or variations in blood sugars) are thought to be responsible. Ketones and ketoacidosis are not direct causes of neuropathies.

A newly admitted patient with type 1 diabetes asks the nurse what caused her diabetes. When the nurse is explaining to the patient the etiology of type 1 diabetes, what process should the nurse describe? A) The tissues in your body are resistant to the action of insulin, making the glucose levels in your blood increase. B) Damage to your pancreas causes an increase in the amount of glucose that it releases, and there is not enough insulin to control it. C) The amount of glucose that your body makes overwhelms your pancreas and decreases your production of insulin. D) Destruction of special cells in the pancreas causes a decrease in insulin production. Glucose levels rise because insulin normally breaks it down.

Ans: D Destruction of special cells in the pancreas causes a decrease in insulin production. Glucose levels rise because insulin normally breaks it down. Feedback: Type 1 diabetes is characterized by the destruction of pancreatic beta cells, resulting in decreased insulin production, unchecked glucose production by the liver, and fasting hyperglycemia. Also, glucose derived from food cannot be stored in the liver and remains circulating in the blood, which leads to postprandial hyperglycemia. Type 2 diabetes involves insulin resistance and impaired insulin secretion. The body does not make glucose.

An older adult patient with type 2 diabetes is brought to the emergency department by his daughter. The patient is found to have a blood glucose level of 623 mg/dL. The patients daughter reports that the patient recently had a gastrointestinal virus and has been confused for the last 3 hours. The diagnosis of hyperglycemic hyperosmolar syndrome (HHS) is made. What nursing action would be a priority? A) Administration of antihypertensive medications B) Administering sodium bicarbonate intravenously C) Reversing acidosis by administering insulin D) Fluid and electrolyte replacement

Ans: D Fluid and electrolyte replacement Feedback: The overall approach to HHS includes fluid replacement, correction of electrolyte imbalances, and insulin administration. Antihypertensive medications are not indicated, as hypotension generally accompanies HHS due to dehydration. Sodium bicarbonate is not administered to patients with HHS, as their plasma bicarbonate level is usually normal. Insulin administration plays a less important role in the treatment of HHS because it is not needed for reversal of acidosis, as in diabetic ketoacidosis (DKA).

A school nurse is teaching a group of high school students about risk factors for diabetes. Which of the following actions has the greatest potential to reduce an individuals risk for developing diabetes? A) Have blood glucose levels checked annually. B) Stop using tobacco in any form. C) Undergo eye examinations regularly. D) Lose weight, if obese.

Ans: D Lose weight, if obese. Feedback: Obesity is a major modifiable risk factor for diabetes. Smoking is not a direct risk factor for the disease. Eye examinations are necessary for persons who have been diagnosed with diabetes, but they do not screen for the disease or prevent it. Similarly, blood glucose checks do not prevent the diabetes.

A nurse is teaching basic survival skills to a patient newly diagnosed with type 1 diabetes. What topic should the nurse address? A) Signs and symptoms of diabetic nephropathy B) Management of diabetic ketoacidosis C) Effects of surgery and pregnancy on blood sugar levels D) Recognition of hypoglycemia and hyperglycemia

Ans: D Recognition of hypoglycemia and hyperglycemia Feedback: It is imperative that newly diagnosed patients know the signs and symptoms and management of hypo- and hyperglycemia. The other listed topics are valid points for education, but are not components of the patients immediate survival skills following a new diagnosis.

A patient with type 2 diabetes has been managing his blood glucose levels using diet and metformin (Glucophage). Following an ordered increase in the patients daily dose of metformin, the nurse should prioritize which of the following assessments? A) Monitoring the patients neutrophil levels B) Assessing the patient for signs of impaired liver function C) Monitoring the patients level of consciousness and behavior D) Reviewing the patients creatinine and BUN levels

Ans: D Reviewing the patients creatinine and BUN levels Feedback: Metformin has the potential to be nephrotoxic; consequently, the nurse should monitor the patients renal function. This drug does not typically affect patients neutrophils, liver function, or cognition.

A nurse is assessing a patient who has diabetes for the presence of peripheral neuropathy. The nurse should question the patient about what sign or symptom that would suggest the possible development of peripheral neuropathy? a. Persistently cold feet b. Pain that does not respond to analgesia c. Acute pain, unrelieved by rest d. The presence of a tingling sensation

Ans: D The presence of a tingling sensation Feedback: Although approximately half of patients with diabetic neuropathy do not have symptoms, initial symptoms may include paresthesias (prickling, tingling, or heightened sensation) and burning sensations (especially at night). Cold and intense pain are atypical early signs of this complication.

Weight loss and exercise can delay onset of what diabetes?

Prediabetes

What diabetes has little or no endogenous insulin and has a threat of renal, retinal, and neurologic complications?

Type 1 diabetes

What diabetes occurs in adulthood and rarely develops ketosis?

Type 2 diabetes

After teaching a client who is recovering from pancreas transplantation, the nurse assesses the clients understanding. Which statement made by the client indicates a need for additional education? a. If I develop an infection, I should stop taking my corticosteroid. b. If I have pain over the transplant site, I will call the surgeon immediately. c. I should avoid people who are ill or who have an infection. d. I should take my cyclosporine exactly the way I was taught.

a. If I develop an infection, I should stop taking my corticosteroid. Immunosuppressive agents should not be stopped without the consultation of the transplantation physician, even if an infection is present. Stopping immunosuppressive therapy endangers the transplanted organ. The other statements are correct. Pain over the graft site may indicate rejection. Anti-rejection drugs cause immunosuppression, and the client should avoid crowds and people who are ill. Changing the routine of antirejection medications may cause them to not work optimally.

A patient has come to the physician's office after finding out that her blood glucose level was 135 mg/dL. She states that she had not eaten before the test and was told to come and see her physician. She asks the nurse if she has diabetes. What is the most accurate nursing response? a. "Having a fasting serum glucose that high certainly indicates diabetes." b. "That test indicates that we need to perform more tests that are specific for diabetes." c. "How do you feel? Do you have any other signs of diabetes?" d. "Do you have a family history of diabetes, stroke, or heart disease? We need to know before making a diagnosis."

b. "That test indicates that we need to perform more tests that are specific for diabetes." The nurse needs to answer the patient's question in a way that gives information and is not misleading. Although 135 mg/dL is high, a nonpathologic explanation may be found. More tests should be performed to evaluate the patient.

A nurse cares for a client who has diabetes mellitus. The nurse administers 6 units of regular insulin and 10 units of NPH insulin at 0700. At which time should the nurse assess the client for potential problems related to the NPH insulin? a. 0800 b. 1600 c. 2000 d. 2300

b. 1600 Neutral protamine Hagedorn (NPH) is an intermediate-acting insulin with an onset of 1.5 hours, peak of 4 to 12 hours, and duration of action of 22 hours. Checking the client at 0800 would be too soon. Checking the client at 2000 and 2300 would be too late. The nurse should check the client at 1600.

A nurse teaches a client with diabetes mellitus about sick day management. Which statement should the nurse include in this clients teaching? a. When ill, avoid eating or drinking to reduce vomiting and diarrhea. b. Monitor your blood glucose levels at least every 4 hours while sick. c. If vomiting, do not use insulin or take your oral antidiabetic agent. d. Try to continue your prescribed exercise regimen even if you are sick.

b. Monitor your blood glucose levels at least every 4 hours while sick. When ill, the client should monitor his or her blood glucose at least every 4 hours. The client should continue taking the medication regimen while ill. The client should continue to eat and drink as tolerated but should not exercise while sick.

A nurse cares for a client who is diagnosed with acute rejection 2 months after receiving a simultaneous pancreas-kidney transplant. The client states, I was doing so well with my new organs, and the thought of having to go back to living on hemodialysis and taking insulin is so depressing. How should the nurse respond? a. Following the drug regimen more closely would have prevented this. b. One acute rejection episode does not mean that you will lose the new organs. c. Dialysis is a viable treatment option for you and may save your life. d. Since you are on the national registry, you can receive a second transplantation.

b. One acute rejection episode does not mean that you will lose the new organs. An episode of acute rejection does not automatically mean that the client will lose the transplant. Pharmacologic manipulation of host immune responses at this time can limit damage to the organ and allow the graft to be maintained. The other statements either belittle the client or downplay his or her concerns. The client may not be a candidate for additional organ transplantation.

A nurse assesses a client with diabetes mellitus. Which clinical manifestation should alert the nurse to decreased kidney function in this client? a. Urine specific gravity of 1.033 b. Presence of protein in the urine c. Elevated capillary blood glucose level d. Presence of ketone bodies in the urine

b. Presence of protein in the urine Renal dysfunction often occurs in the client with diabetes. Proteinuria is a result of renal dysfunction. Specific gravity is elevated with dehydration. Elevated capillary blood glucose levels and ketones in the urine are consistent with diabetes mellitus but are not specific to renal function.

A nurse develops a dietary plan for a client with diabetes mellitus and new-onset microalbuminuria. Which component of the clients diet should the nurse decrease? a. Carbohydrates b. Proteins c. Fats d. Total calories

b. Proteins Restriction of dietary protein to 0.8 g/kg of body weight per day is recommended for clients with microalbuminuria to delay progression to renal failure. The clients diet does not need to be decreased in carbohydrates, fats, or total calories.

A patient has been admitted with hyperglycemic hyperosmolar nonketotic syndrome (HHNS). The blood glucose level is very high (880 mg/dL) on admission. The physician believes that the condition is the result of large amounts of glucose solutions administered intravenously (IV) during renal dialysis. What should the nurse anticipate that the patient would exhibit? a. Fruity breath and a high level of ketones in her urine b. Severe dehydration and hypernatremia caused by the hyperglycemia c. Exactly the same symptoms and signs as DKA d. Kussmaul respirations, nausea, and vomiting

b. Severe dehydration and hypernatremia caused by the hyperglycemia IV solutions containing glucose bypass the digestive system; consequently, the pancreas is not triggered to release insulin. However, just enough insulin is present to prevent the breakdown of fatty acids and the formation of ketones.

When a patient with type 2 diabetes says, "Why in the world are they looking at my hemoglobin? I thought my problem was with my blood sugar." What should the nurse explain about the level of hemoglobin A1c? a. Shows how a high level of glucose can cause a significant drop in the hemoglobin level b. Shows what the glucose level has done during the past 3 months c. Indicates a true picture of the patient's nutritional state d. Reflects the effect of a high level of glucose on the ability to produce red blood cells (RBCs)

b. Shows what the glucose level has done during the past 3 months By analyzing the amount of glucose bound to the hemoglobin, the level of blood glucose can be evaluated for the past 3 months because the glucose stays bound to the hemoglobin for the life of the RBC.

A patient with type 1 diabetes has an insulin order for NPH insulin, 35 U, to be given at 0700. The patient has also been instructed not to take anything by mouth (NPO) in preparation for laboratory work that will not be drawn until 1000. What action should the nurse implement? a. Give the insulin as ordered. b. Give the insulin with a small snack. c. Inform the charge nurse. d. Hold the insulin until after the blood draw.

d. Hold the insulin until after the blood draw. Holding the insulin to adhere to the NPO order is appropriate. The patient will not be getting food until after the laboratory work; consequently, the insulin will not be needed until then. Giving the insulin as ordered will create a possibility of hypoglycemia before the blood is drawn. Giving a snack to a patient who is NPO is inappropriate.

A nurse assesses a client with diabetes mellitus who self-administers subcutaneous insulin. The nurse notes a spongy, swelling area at the site the client uses most frequently for insulin injection. Which action should the nurse take? a. Apply ice to the site to reduce inflammation. b. Consult the provider for a new administration route. c. Assess the client for other signs of cellulitis. d. Instruct the client to rotate sites for insulin injection.

d. Instruct the client to rotate sites for insulin injection. The clients tissue has been damaged from continuous use of the same site. The client should be educated to rotate sites. The damaged tissue is not caused by cellulitis or any type infection, and applying ice may cause more damage to the tissue. Insulin can only be administered subcutaneously and intravenously. It would not be appropriate or practical to change the administration route.

When teaching a client recently diagnosed with type 1 diabetes mellitus, the client states, I will never be able to stick myself with a needle. How should the nurse respond? a. I can give your injections to you while you are here in the hospital. b. Everyone gets used to giving themselves injections. It really does not hurt. c. Your disease will not be managed properly if you refuse to administer the shots. d. Tell me what it is about the injections that are concerning you.

d. Tell me what it is about the injections that are concerning you. Devote as much teaching time as possible to insulin injection and blood glucose monitoring. Clients with newly diagnosed diabetes are often fearful of giving themselves injections. If the client is worried about giving the injections, it is best to try to find out what specifically is causing the concern, so it can be addressed. Giving the injections for the client does not promote self-care ability. Telling the client that others give themselves injections may cause the client to feel bad. Stating that you don't know another way to manage the disease is dismissive of the clients concerns.

A patient has been admitted to the hospital with the diagnosis of DKA. What vital signs should a nurse anticipate that the patient will exhibit? a. Temperature, 99 F; pulse, 62 beats/min; respirations, 16 breaths/min and shallow b. Temperature, 98.6 F; pulse, 76 beats/min; respirations, 16 breaths/min and deep c. Temperature, 98 F; pulse, 84 beats/min; respirations, 18 breaths/min and shallow d. Temperature, 97.4 F; pulse, 110 beats/min; respirations, 26 breaths/min and deep

d. Temperature, 97.4 F; pulse, 110 beats/min; respirations, 26 breaths/min and deep DKA is caused by the attempt of the body to metabolize fat for energy, which results in an acidotic state. The classic signs of DKA are hypothermia, tachycardia, and Kussmaul respirations (rapid and deep) to blow off the acid ions via respirations. The respirations will have a fruity odor.

A patient newly diagnosed with type 2 diabetes mellitus asks the nurse why she has to take a pill instead of insulin. The nurse explains that in type 2 diabetes mellitus, the body still makes insulin. What other information is pertinent for the nurse to relay? a. Overweight and underactive people cannot simply use the insulin produced. b. Metabolism is slowed in some people, so they have to take a pill to speed up their metabolism. c. Sometimes the autoimmune system works against the action of the insulin. d. The cells become resistant to the action of insulin. Pills are given to increase the sensitivity.

d. The cells become resistant to the action of insulin. Pills are given to increase the sensitivity. Type 2 diabetes mellitus is a disease in which the cells become resistant to the action of insulin and the blood glucose level rises. Oral hyperglycemic agents make the cells more sensitive.


Conjuntos de estudio relacionados

Insurance and Risk Management Midterm

View Set

Hinkle Chapter 30: Management of Patients with Hematologic Neoplasms

View Set